LSAT and Law School Admissions Forum

Get expert LSAT preparation and law school admissions advice from PowerScore Test Preparation.

 Administrator
PowerScore Staff
  • PowerScore Staff
  • Posts: 8917
  • Joined: Feb 02, 2011
|
#27049
Complete Question Explanation
(The complete setup for this game can be found here: lsat/viewtopic.php?t=13972)

The correct answer choice is (E)

This question, as distinguished from question #9, asks for you to consider the listed pieces of mail in each answer in combination, and to determine which combination cannot be addressed to J. As we know from the combination of the first and second rules, when L is not addressed to J, then L must be address to R. When L is addressed to R, then P must be address to J. Answer choice (E) contains neither L or P, which violates this relationship. Consequently, the combination in answer choice (A) cannot occur, and (E) is correct.
 shop.shim@gmail.com
  • Posts: 3
  • Joined: Dec 20, 2014
|
#17884
Hello. I have a trouble understanding question No. 10.

The question is: "Which one of the following CANNOT be a complete and accurate list of the pieces of mail addressed to Jana?"

Q1. How I understand is that 4 answer choices CAN BE complete and accurate list of mail addressed to Jana, 1 answer choice cannot be the complete and accurate list. Now, how can there be more than one complete and accurate list of a scenario?

Q2. I do not understand why (E) is the correct answer, which CANNOT be complete and accurate list...

Thank you.
 Andrew Ash
PowerScore Staff
  • PowerScore Staff
  • Posts: 32
  • Joined: Sep 15, 2014
|
#17887
Hi Shop Shim,

You're correct that this question is unusually worded. In this particular case, the phrase "complete and accurate list" only refers to which pieces of mail are addressed to Jana in a particular scenario. So, for instance, in question 8, answer choice (B), the "complete and accurate list" of mail addressed to Jana is the letter and the magazine.

This question is essentially saying this: pretend that Jana gets the items in each answer choice, and only those items. One of the five scenarios will break the game's rules. Which one is it?

Let's take another look at answer choice (E). Jana gets the flyer, the magazine, and the survey, and nothing else. Georgette can never get the letter, because of the first rule, so the letter therefore has to go to Rini. But the second rule says that if Rini gets the letter, then Jana has to get the postcard. We already said that Jana doesn't get the postcard. So it's impossible for the flyer, the magazine, and the survey to be the "complete and accurate list" of items that Jana receives, which means that this is the correct answer choice.

Best,
Andrew
 shop.shim@gmail.com
  • Posts: 3
  • Joined: Dec 20, 2014
|
#17890
Thank you so much. You made it very clear to me.

Get the most out of your LSAT Prep Plus subscription.

Analyze and track your performance with our Testing and Analytics Package.